Search found 16 matches

by kalpita123

Wed Oct 03, 2012 9:24 am
Forum: I just Beat The GMAT!
Topic: 720 (Q-49, V-39)-Stay cool, you will succeed!
Replies: 7
Views: 4686

Thanks Ian and Mitch!
Makes sense!

by kalpita123

Fri Sep 28, 2012 9:59 am
Forum: Problem Solving
Topic: Ratio Problem
Replies: 9
Views: 2244

Hi Mitch, Just need one clarification! In case of two given routes (here, cycling against the wind and cycling with the wind),we do consider rate and time as inversely proportional to each other when the distance is constant in both cases.Am i correct? Here, the distance is not mentioned. Correct! ...

by kalpita123

Thu Sep 27, 2012 10:55 am
Forum: Problem Solving
Topic: Ratio Problem
Replies: 9
Views: 2244

Hi Mitch,
Just need one clarification! In case of two given routes (here, cycling against the wind and cycling with the wind),we do consider rate and time as inversely proportional to each other when the distance is constant in both cases.Am i correct? Here, the distance is not mentioned.

by kalpita123

Wed Sep 26, 2012 12:43 pm
Forum: Problem Solving
Topic: Ratio Problem
Replies: 9
Views: 2244

But in DS question, we should decide that the condition is true or false, In first condition, we can answer it always 'even', so answer is 'true' but in second condition, I can not answer it always 'even' because it can be a even, almost cases, but it can be 'nothing = neither even nor odd' May I d...

by kalpita123

Mon Sep 17, 2012 3:27 pm
Forum: Data Sufficiency
Topic: OG 13th, DS #111. is xy even?
Replies: 10
Views: 2483

Congrats on your stellar performance in the GMAT, eagleeye !

Very nice & detailed debrief !

Good Luck!

by kalpita123

Wed Sep 05, 2012 11:59 am
Forum: I just Beat The GMAT!
Topic: 770 (Q50,V47): How BEATtheGMAT helped me beat the GMAT!!
Replies: 21
Views: 10408

Congrats! Very nice debrief! Especially the highlighted sentences & "Few General Notes" in your debrief are commendable.

Loads of Luck to you!

by kalpita123

Tue Sep 04, 2012 11:06 pm
Forum: I just Beat The GMAT!
Topic: 720 (Q50, V38, AWA6, IR3) - How I saved my GMAT?
Replies: 10
Views: 7604

I am able to solve the below problem using "number plugging" method,however, looking for some other ways to approach the problem: If a is not equal to b, is [ 1/(a-b)]> ab ? 1) mod a > mod b 2) a<b ans- E Here's a way to do it using mathematical reasoning: First, let's rephrase the expres...

by kalpita123

Tue Jul 31, 2012 1:32 pm
Forum: Data Sufficiency
Topic: MOD+Inequality
Replies: 5
Views: 2967

MOD+Inequality

I am able to solve the below problem using "number plugging" method,however, looking for some other ways to approach the problem:

If a is not equal to b, is [ 1/(a-b)]> ab ?
1) mod a > mod b
2) a<b

ans- E

by kalpita123

Mon Jul 30, 2012 3:32 pm
Forum: Data Sufficiency
Topic: MOD+Inequality
Replies: 5
Views: 2967

I chose B . Candidates make statements that they think are true . >> They think statements are true Vs Candidates make statements that they think to be true . >> They think statements to be true. First one sounds better to me. Btw, what exactly is the difference between these two statements? "...

by kalpita123

Mon Jul 30, 2012 2:52 pm
Forum: Sentence Correction
Topic: question pack1
Replies: 8
Views: 2292

If n is a positive integer between 1 and 99, inclusive, what is the probability that n(n+1) is a multiple of 3? 1) 1/4 2) 1/3 3) 1/2 4) 2/3 5) 5/6 For n(n+1) to be a multiple of 3, either n or n+1 must be a multiple of 3. n, n+1, and n+2 are 3 consecutive integers. Of every 3 consecutive integers, ...

by kalpita123

Tue Jul 24, 2012 1:24 pm
Forum: Problem Solving
Topic: probability
Replies: 11
Views: 5063

if a*b*c= 60, where a,b and c are integers greater than 1, a+b+c=? 1)a+b=8 2)Both a and b are odd numbers. Please help.. Prime factorization comes in handy for this one. 60 = 2x2x3x5 Since a, b and c are integers greater than 1, there are only 3 cases that are possible. case 1: the numbers are 2, 3...

by kalpita123

Sun Jul 15, 2012 3:53 pm
Forum: Data Sufficiency
Topic: HELP
Replies: 3
Views: 1262

In ans choice (B), is the modifier "that" not modifying only "a rooster" ?

Is this an exception to the "touch rule"?

by kalpita123

Thu Jul 12, 2012 12:41 pm
Forum: Sentence Correction
Topic: sc question
Replies: 11
Views: 4213

eagleeye: Thanks for the clarification! You are right, as all vertical lines (including Y-axis) will have undefined slopes.I had assumed the same. Well, i wanted to know whether a line with a negative fractional slope (say= -1/4) ans a -ve y-intercept (say= -5)will pass through the First Quadrant o...

by kalpita123

Tue Jun 26, 2012 7:14 pm
Forum: Data Sufficiency
Topic: GMATPrep pack DS
Replies: 6
Views: 1940

In the XY-Plane , is the slope of the line k equal to 0 ? (1) The X- intercept of k is 0 (2) The y-intercept of k is 0. Just note that any line passing through the origin satisfies both the condition but the line may or may not have slope equal to zero. Hence, both statements together is insufficie...

by kalpita123

Tue Jun 26, 2012 12:30 pm
Forum: Data Sufficiency
Topic: GMATPrep pack DS
Replies: 6
Views: 1940